Ch 42 Gastrointestinal 2 - Upper GI

Lakukan tugas rumah & ujian kamu dengan baik sekarang menggunakan Quizwiz!

Which medication increases lower esophageal sphincter pressure? 1 Diazepam 2 Bethanechol 3 Theophylline 4 Morphine sulfate

2 Bethanechol increases the pressure in the lower esophagus. Diazepam, theophylline, and morphine sulfate are medications that decrease lower esophageal pressure. Text Reference - p. 934

The patient history indicates the patient was taking ondansetron at home before admission. The nurse inquires as to the effectiveness of this medication in treating which symptom? 1 Headache 2 Nausea 3 Pain 4 Leg cramps

2 Ondansetron is an antiemetic. The nurse would inquire as to its effectiveness in reducing the patient's nausea. Ondansetron will not treat headaches, pain, or leg cramps. Text Reference - p. 926

Which pathophysiologic condition is associated with duodenal ulcers? 1 Shock 2 Sepsis 3 Cirrhosis 4 Large hiatal hernia

3 Cirrhosis and pancreatitis are the pathophysiologic conditions associated with duodenal ulcers. Shock, sepsis, and large hiatal hernia are conditions that cause gastritis. Text Reference - p. 944

Which microorganism causes Vincent's infection? 1 Staphylococcus 2 Candida albicans 3 Fusiform bacteria 4 Herpes simplex virus

3 Fusiform bacteria cause Vincent's infection. Staphylococcus species cause parotitis. Oral candidiasis is caused by Candida albicans. Herpes simplex virus causes herpes. Text Reference - p. 928

Which enzyme activates the immune response with antibody production and the release of inflammatory cytokines? 1 Pepsin 2 Urease 3 Bile salts 4 Serum amylase

2 Urease activates the immune response with body antibody production and the release of inflammatory cytokines, resulting in mucosal edema. Pepsin is an enzyme released in the stomach that degrades food particles into peptides. Bile salts are released by liver cells, which bind with cholestyramine and cause gastric irritation. Serum amylase levels determine pancreatic function. Text Reference - p. 946

The primary health care provider orders endoscopic nutrition via a nasogastric tube for a postsurgical patient whose nutrition is disrupted. Which interventions should the nurse perform for this patient? Select all that apply. 1 Irrigating the tube 2 Providing oral care 3 Providing nasal care 4 Assessing the drainage 5 Repositioning the tube

2 3 4 While providing care for a postsurgical patient with a nasogastric tube, the nurse should provide oral and nasal care to the patient. The nursing responsibilities also include assessing the drainage from the nasogastric tube. Irrigating and repositioning the tube should be performed only after consulting the primary health care provider. Text Reference - p. 939

The patient has a prescription for metoclopramide 10 mg intravenously (IV). Available is a premixed IV bag containing 40 mg/50 mL. How many mL of this solution should be infused? 1 1.25 mL 2 3.75 mL 3 12.5 mL 4 37.5 mL

3 Using ratio and proportion, multiply 40 by x and multiply 10 × 50 to yield 40x = 500. Divide 500 by 40 to yield 12.5 mL. Text Reference - p. 926

The nurse is caring for a patient with anorexia, nausea, vomiting, epigastric pain, and feeling of fullness. Which interventions are beneficial to the patient? Select all that apply. 1 Inserting a nasogastric tube 2 Administering intravenous fluids 3 Maintaining the patient on nothing-by-mouth (NPO) status 4 Educating the patient that fainting may occur due to cimetidine 5 Providing the patient with nonirritating diet consisting of six small feedings a day

1 2 3 Anorexia, nausea, vomiting, epigastric pain, and a feeling of fullness are the symptoms of acute gastritis. Therefore, a nasogastric tube should be inserted to irrigate the precipitating agent from the stomach, to monitor bleeding, and to keep the stomach free from noxious smell. Administration of intravenous fluids may compensate for fluid loss in patients due to vomiting. The patient with acute gastritis should be kept on nothing-by-mouth (NPO) status to prevent vomiting. Drug therapy for gastritis includes H2-receptor blockers (ranitidine, cimetidine) and proton pump inhibitors (lansaprazole, omiprazole). The nurse should inform the patient about the therapeutic effects of the medications. Cimetidine is an H2-receptor blocker that causes headache, abdominal pain, constipation, and diarrhea; it is not associated with fainting. Providing a nonirritating diet consisting of six small feedings per day is helpful for patients with chronic gastritis. Text Reference - p. 942

Which laboratory results determine the type and amount of intravenous fluids administered? Select all that apply. 1 Hematocrit 2 Electrolytes 3 Hemoglobin 4 Liver enzymes 5 Serum amylase

1 2 3 Hematocrit, electrolytes, and hemoglobin are laboratory test results that determine the type and amount of intravenous fluids administered based on the degree of dehydration, vomiting, and electrolyte imbalance. Liver enzymes are used to detect any liver problems that may complicate ulcer healing. Serum amylase helps in determining pancreatic function. Text Reference - p. 948

A nurse is teaching an obese patient with gastroesophageal reflux disease (GERD) measures that should be taken to prevent complications. What instructions should the nurse give? Select all that apply. 1 Maintain a low-fat diet. 2 Avoid smoking cigarettes. 3 Use anticholinergic drugs, as prescribed. 4 Avoid tea and coffee. 5 Lie down immediately after having food.

1 2 4 In an obese person, the intraabdominal pressure is increased, which can exacerbate GERD. Maintaining a low-fat diet could help in losing weight and therefore relieve the condition. Tea, coffee, and nicotine (a component of cigarettes) are known to decrease the lower esophageal sphincter pressure, aggravating GERD. Patients with GERD are prescribed cholinergic drugs to relieve their condition. Anticholinergic drugs, on the other hand, affect the lower esophageal sphincter pressure and may therefore cause GERD. Lying down immediately after eating food may promote the movement of food toward the esophageal sphincter and increase the pressure on it, therefore exacerbating the condition. Text Reference - p. 933

While providing fluids to a postgastrectomy patient, which symptoms support the nurse's suspicion that there is leakage from a jejunostomy feeding tube? Select all that apply. 1 Pain 2 Fever 3 Cough 4 Regurgitation 5 Difficulty breathing

1 2 5 When there is a leakage from the feeding tube, the patient experiences pain, high temperature, and dyspnea (difficulty breathing). Therefore, these symptoms support the nurse's suspicion. Cough and regurgitation are manifestations of esophageal cancer. Text Reference - p. 938

Which medications increase the risk of ulcer development? Select all that apply. 1 Aspirin 2 Fluoxetine 3 Misoprostol 4 Bethanechol 5 Metoclopramide

1 2 Aspirin and fluoxetine are ulcerogenic drugs that inhibit the synthesis of prostaglandins, increase gastric acid secretion, and reduce the integrity of the mucosal barrier. These medications increase the risk of ulcer development. Misoprostol, bethanechol, and metoclopramide are used in gastroesophageal reflux disease and peptic ulcer drug therapy. Text Reference - p. 943

The nurse provides information to a student nurse about appropriate goals for patients with upper gastrointestinal bleeding. The identification of which goals by the student nurse indicates the need for further teaching? Select all that apply. 1 Decreased hemoglobin level 2 Optimal nutritional status 3 Absence of gastrointestinal bleeding 4 Hemodynamic stability 5 Minimal symptoms of pain or anxiety

1 2 The initial hemoglobin in patients with acute upper GI bleeding will often be at the patient's baseline because the patient is losing whole blood. With time (typically after 24 hours or more) the hemoglobin will decline as the blood is diluted by the influx of extravascular fluid into the vascular space and by fluid administered during resuscitation. Decreased hemoglobin is not an appropriate goal. Optimal nutritional status is a goal for patients with stomach cancer. The goals for patients with upper gastrointestinal bleeding include no gastrointestinal bleeding, a return to a normal hemodynamic state, and minimization of pain or anxiety.

A patient presents with suspected upper gastrointestinal bleeding. Which questions are appropriate for the nurse to ask to assess the patient's cognitive-perceptual pattern? Select all that apply. 1 "Do you have abdominal cramps?" 2 "Do you experience any epigastric pain?" 3 "Do you have a family history of bleeding?" 4 "Do you experience weakness and fainting?" 5 "Do you experience acute or chronic stress?"

1 2 While assessing the effects of upper gastrointestinal bleeding on a patient's cognitive-perceptual pattern, the appropriate question to ask is whether the patient has experienced any abdominal cramps and epigastric pain. The answers help determine the effect on the nociceptor that sends signals to the brain and spinal cord. Asking the patient about his or her family history of bleeding will determine the past health history. Obtaining information from the patient about weakness and fainting will determine the activity exercise pattern. Asking the patient about any experience of acute or chronic stress will determine the patient's coping-stress tolerance pattern. Text Reference - p. 955

What are the characteristics of gastric ulcers? Select all that apply. 1 Gastric ulcer lesions are superficial. 2 Gastric ulcers increase the gastric secretion. 3 Gastric ulcers predominantly occur in antrum. 4 Gastric ulcers cause pain one to two hours after meals. 5 Gastric ulcers manifest as burning, cramping pain in the midepigastrium.

1 3 4 Gastric ulcer lesions are superficial, round, oval, or cone shaped. They predominantly occur in antrum and cause pain one to two hours after meals. Gastric ulcers decrease gastric secretion, whereas duodenal ulcers increase gastric secretion. In gastric ulcers, there is a burning, cramping pain in the high left epigastrium. Text Reference - p. 943

A patient has had esophageal surgery, and a jejunostomy feeding tube is inserted to administer oral fluids. The nurse has been told to check for signs of intolerance and leakage of feeding into the mediastinum. Which signs should the nurse be observant for? Select all that apply. 1 Pain 2 Acid reflux 3 Dyspnea 4 Tachycardia 5 Increased temperature

1 3 5 With tube feedings, the patient should be observed for signs of intolerance of feeding or leakage of the feeding into the mediastinum. Symptoms that indicate leakage are pain, dyspnea, and increased temperature. Feeding is done through the tube in an upright position; therefore, the chance of acid reflux is unlikely. Tachycardia doesn't occur immediately as a sign of leakage into the mediastinum. Text Reference - p. 938

Which medications can be co-administered with antacids to provide ulcer relief? Select all that apply. 1 Cimetidine 2 Amoxicillin 3 Famotidine 4 Omeprazole 5 Clarithromycin

1 3 Cimetidine and famotidine are histamine-receptor blockers that can be concurrently used with antacids like aluminum hydroxide, because it neutralizes the acid to provide relief. Amoxicillin is used in triple-drug therapy to treat Helicobacter pylori (H. pylori) infection. Along with antibiotics, omeprazole reduces gastric secretion. Clarithromycin is included in antibiotic therapy to treat H. pylori infection. Text Reference - p. 945

Which complications can occur due to long-term use of proton pump inhibitors in patients with gastroesophageal reflux disease (GERD)? Select all that apply. 1 Pneumonia 2 Renal calculi 3 Uterine rupture 4 Decreased bone density 5 Chronic hypochlorhydria

1 4 5 Long-term use of proton pump inhibitors is associated with pneumonia, decreased bone density, and chronic hypochlorhydria. Renal calculi are associated with antacids. Prostaglandins can cause uterine rupture. Text Reference - p. 934

The nurse is caring for a patient who is on misoprostol medication for peptic ulcer disease. Which side effects does the nurse anticipate in this patient? Select all that apply. 1 Diarrhea 2 Tachycardia 3 Blurred vision 4 Uterine cramping 5 Abdominal cramp

1 4 5 Misoprostol is a synthetic prostaglandin analog that can cause diarrhea, uterine cramping, and abdominal cramping. Tachycardia and blurred vision are the side effects of anticholinergics. Text Reference - p. 947

A nurse is teaching a patient about prevention of peptic ulcers. What instructions should the nurse give the patient? Select all that apply. 1 Avoid smoking. 2 Consume raw uncooked food. 3 Use nonsteroidal antiinflammatory drugs (NSAIDs) for treatment of pain. 4 Wash hands thoroughly with soap after using the restroom and before eating. 5 Report symptoms of gastric irritation, such as nausea and epigastric pain, to the health care provider.

1 4 5 Nicotine, a component of cigarettes, causes gastric irritation, and therefore smoking should be avoided by those with peptic ulcers. Washing hands thoroughly with soap after using the restroom and before eating would help prevent the Helicobacter pylori infection that causes peptic ulcers. Any symptom of gastric irritation such as nausea and epigastric pain must be reported to the health care provider to prevent lethal consequences of peptic ulcer disease. Consumption of raw uncooked food increases the chance of H. pylori infection; therefore, it should be avoided. NSAIDs should not be taken over a long period of time, because they are a potent gastric irritant. Text Reference - p. 946

Which otolaryngologic symptoms occur in gastroesophageal reflux disease (GERD)? Select all that apply. 1 Choking 2 Dyspnea 3 Wheezing 4 Hoarseness 5 Sore throat

1 4 5 Otolaryngologic symptoms in GERD include choking, hoarseness, and sore throat. Wheezing and dyspnea are the respiratory symptoms that occur in GERD. Text Reference - p. 932

The nurse is teaching care management to a patient with gastroesophageal reflux disease (GERD). In the follow-up visit, the patient complains of severe heartburn. Which actions indicate the need for further teaching? Select all that apply. 1 The patient eats oranges daily. 2 The patient drinks 2 L of water daily. 3 The patient chews gum daily. 4 The patient eats ice cream often. 5 The patient eats ginger daily. 6 The patient drinks a cup of milk at bedtime.

1 4 6 Oranges are a source of citric acid. Eating acidic foods aggravates the symptoms of gastroesophageal reflux disease (GERD). Ice cream is rich in fatty acids. Fats tend to decrease lower esophageal sphincter (LES) pressure, resulting in regurgitation of stomach acid. Drinking a cup of milk at bedtime increases gastric acid secretion. Therefore, the nurse recommends that the patient avoid oranges, ice cream, and milk. Drinking 2 L of water neutralizes the pH of stomach acid and reduces the symptoms of GERD. Chewing gum increases the production of saliva, thereby helping neutralize the pH of gastric acid. Ginger is known for its antiinflammatory and antacid activities. Text Reference - p. 933

The nurse is preparing a care plan for a patient who underwent an open high abdominal incision. Which necessary interventions should the nurse include in the care plan? Select all that apply. 1 Monitoring pulse rate 2 Monitoring bowel sounds 3 Monitoring blood glucose 4 Monitoring respiratory rate 5 Monitoring serum creatinine

1 4 After an open high abdominal incision, respiratory complications can occur. Therefore, the nurse should monitor pulse rate and respiratory rate. Bowel sounds and blood glucose level are not altered after an open high abdominal incision. Serum creatinine levels are altered in kidney and liver diseases but not after an open high abdominal incision. Text Reference - p. 935

The nurse is preparing to administer a dose of octreotide to a patient who is experiencing an acute episode of upper gastrointestinal (GI) bleeding and is waiting for an endoscopy procedure. The nurse knows that the octreotide will have which mechanism of action? Select all that apply. 1 Decreases blood flow to the GI tract. 2 Increases the production of mucus in the stomach. 3 Causes tissue edema and pressure on the source of bleeding. 4 Decreases hydrochloric acid secretion by decreasing the release of gastrin. 5 Causes vasoconstriction, reducing pressure in the portal circulation and stopping bleeding.

1 4 Octreotide is a somatostatin analog that works by reducing blood flow to the GI tract and reduces hydrochloric acid secretion by reducing the release of gastrin. Octreotide does not increase production of mucus in the stomach. Epinephrine, when injected during an endoscopy procedure, produces hemostasis by causing tissue edema and pressure on the source of bleeding. Vasopressin works by causing vasoconstriction, reducing pressure in the portal circulation and stopping the bleeding. Text Reference - p. 955

A patient with a history of peptic ulcer disease has presented to the emergency department reporting severe abdominal pain and has a rigid, board-like abdomen that prompts the health care team to suspect a perforated ulcer. What intervention should the nurse anticipate? 1 Providing intravenous (IV) fluids and inserting a nasogastric (NG) tube 2 Administering oral bicarbonate and testing the patient's gastric pH level 3 Performing a fecal occult blood test and administering IV calcium gluconate 4 Starting parenteral nutrition and placing the patient in a high-Fowler's position

1 A perforated peptic ulcer requires IV replacement of fluid losses and continued gastric aspiration by NG tube. Oral bicarbonate would not be given, because the client would be nothing by mouth, and gastric pH testing is not a priority. Calcium gluconate is not a medication directly relevant to the patient's suspected diagnosis and parenteral nutrition is not a priority in the short term. Text Reference - p. 947

Which medication causes gastritis? 1 Aspirin 2 Amoxicillin 3 Lansoprazole 4 Metronidazole

1 Aspirin is a nonsteroidal antiinflammatory drug (NSAID) that causes gastritis by decreasing prostaglandin in the lining of the stomach. Amoxicillin is used in the treatment of H. pylori infections. Lansoprazole and metronidazole are used in the treatment of gastritis. Text Reference - p. 941

A patient undergoes gastrectomy. What should the nurse recommend to decrease the symptoms of dumping syndrome? 1 Drinking no fluids with meals 2 Remaining in a high Fowler's position after meals 3 Increasing the carbohydrate intake with each meal 4 Consuming a large amount of water with each meal

1 Avoiding fluids with meals prevents dilution and liquefaction of food and thus slows the movement of food into the jejunum. Postgastrectomy patients are often instructed to eat "dry" meals. Remaining in a high Fowler's position after meals may increase the risk for dumping syndrome. A diet high in carbohydrates, especially simple carbohydrates, increases the risk of dumping syndrome. Taking fluids with meals causes stomach contents to empty more rapidly into the jejunum, resulting in dumping syndrome. Text Reference - p. 949

Which side effect occurs in a patient with gastroesophageal reflux disease who is on bethanechol therapy? 1 Syncope 2 Flatulence 3 Renal calculi 4 Hallucinations

1 Bethanechol is a cholinergic medication used to treat gastroesophageal reflux disease. Cholinergics cause syncope, which is the partial or complete loss of consciousness. Proton pump inhibitors cause flatulence. Renal calculi can occur with antacid use. Prokinetic medications cause hallucinations. Text Reference - p. 934

Assessment findings of a patient include a reflux of bile and pancreatic secretions and clinical manifestations of shock. The nurse suspects what diagnosis? 1 Gastritis 2 Achalasia 3 Stomach cancer 4 Upper gastrointestinal bleeding

1 Burns, reflux of bile and pancreatic secretions, and shock are the clinical manifestations of gastritis. Achalasia is characterized by substernal chest pain, dysphasia, and nocturnal regurgitation. Early satiety, abdominal pain, and weight loss are the clinical manifestations of stomach cancer. Bloody vomitus, and black, tarry stools are manifestations of upper gastrointestinal bleeding. Text Reference - p. 941

A patient with gastroesophageal reflux disease (GERD) is on cimetidine therapy. Which parameter does the nurse monitor to provide effective care? 1 Bowel sounds 2 Motor movements 3 Serum calcium levels 4 Serum magnesium levels

1 Cimetidine is a histamine-receptor used in the treatment of peptic ulcer and GERD. Cimetidine decreases gastric motility and causes constipation. Therefore, the nurse monitors the patient for any changes in bowel sounds. This may help with early detection of constipation. Cimetidine does not have extrapyramidal side effects and does not cause motor incoordination. Cimetidine does not alter serum calcium or magnesium levels. Text Reference - p. 934

Which condition results from sympathetic activation? 1 Diaphoresis 2 Increase in salivation 3 Increase in gastric motility 4 Relaxation of lower esophageal sphincter

1 Excessive sweating indicates diaphoresis. Sympathetic activation causes diaphoresis. Parasympathetic stimulation increases salivation, gastric motility, and relaxes the lower esophageal sphincter. Text Reference - p. 925

To prevent the recurrence of gastritis, what instruction should the nurse provide to the patient? 1 Stop smoking and do not consume alcoholic beverages. 2 Consume a soft diet with moderate spices and seasonings. 3 Stop smoking and consume alcoholic beverages in moderation. 4 Take acetaminophen and ibuprofen for pain.

1 Lifestyle modifications to prevent the recurrence of gastritis include refraining from consuming alcohol and cigarette smoking. Alcohol directly irritates the gastric mucosa, and nicotine from cigarette smoke inhibits neutralization of gastric acid. A bland diet (reduced spices and seasonings) is recommended to prevent the recurrence of gastritis. Alcohol should be eliminated from the diet, not merely reduced. Advising the patient to take acetaminophen and ibuprofen (Motrin) is incorrect, because ibuprofen should not be taken by the gastritis patient. Text Reference - p. 941

A 65-year-old female patient who has been on warfarin for five years reports nausea, episodes of vomiting, and pain in the abdomen upon palpation. What could be the reason for the patient's condition? 1 Gastritis 2 Oral cancer 3 Stomach cancer 4 Peptic ulcer disease

1 Nausea, vomiting, and abdominal pain are the characteristics of gastritis. Warfarin is an anticoagulant, which causes gastritis by increasing the risk of bleeding. Oral cancer is characterized by sore throat, sore mouth, and voice changes. Stomach cancer involves weight loss, anemia, weakness, dizziness, and supraclavicular lymph nodes. Peptic ulcer disease includes ulcers; this is characterized by experiencing burning pains in the abdomen after eating.Text Reference - p. 941

Which histamine-receptor blocker is available only for oral administration? 1 Nizatidine 2 Ranitidine 3 Cimetidine 4 Famotidine

1 Nizatidine is available only for oral administration but not for intravenous (IV) administration; the medication is used to promote ulcer healing. Ranitidine, cimetidine, and famotidine are histamine (H2) blockers that can be given orally or through IV. Text Reference - p. 946

Which condition is associated with symptoms of weight loss, halitosis, nocturnal regurgitation, and an inability to belch? 1 Achalasia 2 Esophageal varices 3 Esophageal stricture 4 Eosinophilic esophagitis (EE)

1 Nocturnal regurgitation, an inability to belch, weight loss, and halitosis may indicate achalasia. Achalasia is the absence of peristalsis in the lower two-thirds of the esophagus. Esophageal varices are dilated, tortuous veins that occur in the lower portion of the esophagus. Esophageal stricture is the narrowing of the esophagus. EE is the inflammation of the esophagus due to an infiltration of eosinophils. Text Reference - p. 940

The nurse is caring for a patient who underwent surgical therapy for peptic ulcer disease. Upon clinical examination, the primary health care provider suspected that a complication occurred due to loss of intrinsic factor. Which complication does the nurse suspect? 1 Pernicious anemia 2 Bile reflux gastritis 3 Dumping syndrome 4 Postprandial hypoglycemia

1 Pernicious anemia is a complication that occurs in a patient after undergoing a partial gastrectomy. It occurs due to loss of intrinsic factor, which is required for the production of vitamin B12. Bile reflux gastritis occurs as a complication of gastric surgery of the pylorus. Dumping syndrome occurs as a complication after surgical removal of the stomach and pylorus. Postprandial hypoglycemia is a variant of dumping syndrome that occurs due to uncontrolled gastric emptying of a bolus of fluid high in carbohydrate into the small intestine. Text Reference - p. 950

Which laboratory test is beneficial for diagnosing the presence of a small amount of blood in gastric secretions and stools? 1 Guaiac test 2 Liver enzyme studies 3 Complete blood count (CBC) 4 Serum amylase determination

1 Small amounts of blood in gastric secretions and stools indicate occult blood, and the guaiac test is used to detect traces of blood in the feces. Liver enzyme studies are performed to detect any liver problems. A complete blood count (CBC) test is used to detect anemia, which is secondary to ulcer bleeding. Serum amylase determination helps to determine pancreatic function. Text Reference - p. 954

A patient presents with complaints of abdominal cramping and diarrhea, which initially was watery but is now bloody. Symptoms began three days ago after eating out with friends. You suspect the patient is experiencing 1 Escherichia coli O157:H7 infection 2 Staphylococcal toxin 3 Botulism 4 Clostridial

1 The clinical manifestations of E. coli O157:H7 include diarrhea (often bloody) and abdominal cramping pain for two to eight days (average three to four days) after swallowing the organism. The diarrhea may start out as watery but may progress to bloody. Stapylococcal toxin has a 30 minute to seven hour onset with nausea, abdominal cramping, and diarrhea as its clinical manifestations. Botulism has an onset of 12 to 36 hours with clinical manifestations of nausea, vomiting, abdominal pain, constipation, distention, and central nervous system alterations. Clostridial has an 8 to 24 hour onset with diarrhea, nausea, abdominal cramping, vomiting (rare), and midepigastric pain as clinical manifestations. Text Reference - p. 957

The nurse is interviewing a patient with a duodenal ulcer. Which characteristic of pain is the nurse likely to find? 1 The pain is cramp like. 2 The pain occurs one to two hours after a meal. 3 The pain is located high up in the epigastrium. 4 The pain is aggravated by food intake.

1 The pain related to a duodenal ulcer is cramp like and appears five to six hours after a meal. The pain is located in the midepigastric region beneath the xiphoid process. The pain is relieved by food intake. Text Reference - p. 942

The nurse is teaching care guidelines to the caregiver of a patient with upper gastrointestinal (GI) bleeding. In the follow-up visit, the patient complains of traces of blood in the vomit. Which action of the patient's caregiver is responsible for the patient's condition? 1 The caregiver gave aspirin to the patient on an empty stomach. 2 The caregiver allowed the patient to drink 10 mL of alcohol once a month. 3 The caregiver gave the patient an analgesic along with a proton pump inhibitor. 4 The caregiver encouraged the patient to practice deep breathing exercises.

1 Traces of blood in the vomit indicate gastrointestinal bleeding. When given on an empty stomach, aspirin irritates the gastrointestinal mucosa and causes gastrointestinal (GI) bleeding. Therefore, it should be taken along with meals or snacks. Alcohol consumption aggravates the symptoms of hyperacidity. However, drinking 10 mL of alcohol once a month will not cause GI bleeding. Co-administration of nonsteroidal antiinflammatory drugs (NSAIDs) and proton-pump inhibitors reduces the risk of bleeding. Deep breathing provides relaxation and does not cause GI bleeding. Text Reference - p. 956

Which dietary instructions should the nurse provide the caregiver of a postoperative patient with dumping syndrome? Select all that apply. 1 "Avoid giving cheese." 2 "Give the patient eggs and meat." 3 "Avoid giving jelly and jam." 4 "Avoid giving fluids with meals." 5 "Divide the meals into three feedings."

2 3 4 Eggs and meat contain proteins and fat, which help to rebuild body tissues and meet energy demands. Distention and fullness of the stomach can occur if fluids are consumed along with meals. Jelly and jam cause diarrhea and dizziness; these foods should be avoided. Cheese contains proteins and fats and should be provided to the patient. The meals of the patient should be divided into six small feedings to avoid overloading the stomach and intestine during meal times. Text Reference - p. 950

A patient complains of burning pain in the abdomen four hours after eating a meal. Upon assessment, the nurse identifies the pain in midepigastric region. Which risk factors are associated with the pain? Select all that apply. 1 Bile reflux 2 Chronic pancreatitis 3 Chronic kidney disease 4 Zollinger-Ellison syndrome 5 Incompetent pyloric sphincter

2 3 4 Patients with duodenal ulcers experience burning or cramp-like pain in the midepigastric region beneath the xiphoid process, generally 2 to 4 hours after eating. The risk factors associated with duodenal ulcers are chronic pancreatitis, chronic kidney disease, and Zollinger-Ellison syndrome. Bile reflux and incompetent pyloric sphincter are the risk factors for gastric ulcers. Text Reference - p. 944

The nurse is caring for a patient diagnosed with Helicobacter pylori (H. pylori) infection. Which medications should the nurse expect to be included in the triple-drug therapy? Select all that apply. 1 Cimetidine 2 Amoxicillin 3 Omeprazole 4 Tetracycline 5 Clarithromycin

2 3 5 Amoxicillin, omeprazole, and clarithromycin are the medications included in triple-drug therapy to treat Helicobacter pylori (H. pylori) infection. Cimetidine is an H2-receptor blocker that is used to reduce gastric hydrochloric acid secretion. Tetracycline is an antibiotic that is included in quadruple therapy for the treatment of H. pylori infection. Text Reference - p. 942

A 54-year-old patient admitted with cancer has not been able to eat because of nausea. Which strategies should the nurse use to increase the patient's intake? Select all that apply. 1 Serve foods that are warm to hot in temperature. 2 Administer antiemetics one hour before meals to prevent nausea. 3 Offer the patient foods, such as cooked cereal and soft or canned fruits. 4 Offer the patient meats and foods with mild spices. 5 Offer a diet that appeals to the patient's preferences.

2 3 5 Foods such as cooked cereals, soft or canned fruits, crackers, toast, and sherbet are high-carbohydrate, low-fat foods that are helpful in managing nausea. An antiemetic before meals also would decrease nausea. Planning the diet around the patient's preferences also will help stimulate the appetite. Hot foods and foods with spices may worsen nausea. Text Reference - p. 927

The nurse is caring for a patient with a hiatal hernia. Which instructions should the nurse include teaching? Select all that apply. 1 "You may drink soda." 2 "You should avoid caffeine." 3 "You should avoid chocolate." 4 "You should suck peppermint." 5 "You should drink orange juice."

2 3 Caffeine and chocolate are reflux-inducing foods that irritate the esophagus or weaken the lower esophageal sphincter causing backward flow of stomach contents. Therefore, caffeine and chocolate should be avoided in patients with a hiatal hernia. Soda is acidic and should be avoided in patients with a hiatal hernia. Peppermint is a reflux-inducing food and should be avoided. Acidic pH beverages such as orange juice should also be avoided by patients with a hiatal hernia. Text Reference - p. 936

After assessing a patient with gastrointestinal bleeding, the nurse suspects shock in the patient. Which findings in the patient support the nurse's conclusion? Select all that apply. 1 Dry skin 2 Increased thirst 3 Rapid, weak pulse 4 High blood pressure 5 Increased temperature

2 3 Shock is a life-threatening condition involving low blood perfusion to the tissues. The circulatory volume is reduced during shock, resulting in the release of antidiuretic hormone (ADH). This helps to increase the renal reabsorption, thereby increasing the blood volume. As a result, thirst is increased. The patient has a rapid, weak pulse due to a low oxygen supply to the tissues. The patient with shock has clammy skin, low blood pressure, and a temperature lower than the normal body temperature. Text Reference - p. 956

The nurse finds that the patient has a hard, painless ulcer on the upper lip. What treatment does the nurse expect the primary health care provider to prescribe? 1 Glossectomy 2 Surgical excision 3 Mandibulectomy 4 Hemiglossectomy

2 A hard, painless ulcer is a sign of oral cancer affecting the lip. Surgical excision of the ulcer is the treatment option for cancer of the lip. Glossectomy is the complete removal of the tongue. It is performed when the tongue becomes cancerous. Mandibulectomy is the removal of the mandible (lower jaw). It is done when the mandibular region is affected by oral cancer. Hemiglossectomy is the surgical removal of part of the tongue. Hemiglossectomy is prescribed if the patient has tumors on part of the tongue. Text Reference - p. 929

A patient presents with suspected gastric carcinoma. The nurse anticipates that what diagnostic test will be prescribed? 1 Radiology 2 Tissue biopsy 3 Rapid urease testing 4 Endoscopic ultrasound

2 A tissue biopsy involves examining tissue removed from the stomach to detect gastric carcinoma. Radiology is not the best method because superficial mucosa is generally involved and changes will not show up clearly on x-ray examination. Rapid urease testing is used to detect the presence of urease in Helicobacter pylori infections. An endoscopic ultrasound is used to stage the disease of stomach cancer. Text Reference - p. 941

Which statement made by the nurse to an achalasia patient about how to manage symptoms needs correction? 1 "You should eat slowly." 2 "You should only eat soft food." 3 "You should drink fluids with meals." 4 "You should sleep with your head elevated."

2 Achalasia is a condition in which peristalsis of the lower two-thirds of the esophagus is absent. Patients with achalasia can also eat semisoft foods. The patient should eat food slowly. The patient should drink fluids with meals and should keep the head elevated while sleeping. Text Reference - p. 940

The nurse is teaching about home care to a patient with gastrointestinal reflux disease (GERD). Which statement made by the patient indicates the need for further teaching? 1 "I should not bend over after eating." 2 "I should lie down for two to three hours after eating." 3 "I should avoid eating within three hours of bedtime." 4 "I should sleep with the head of the bed elevated."

2 After eating, the patient should wait for three hours to lie down. This will help maintain gravity, which in turn prevents the development of acid reflux. The patient should not bend over after eating. The patient should not eat food within three hours of bedtime. The head of the bed should be elevated during sleep. Text Reference - p. 933

The results of a patient's recent endoscopy indicate the presence of peptic ulcer disease (PUD). Which teaching point should the nurse provide to the patient based on this new diagnosis? 1 "You'll need to drink at least two to three glasses of milk daily." 2 "It would likely be beneficial for you to eliminate drinking alcohol." 3 "Many people find that a minced or pureed diet eases their symptoms of PUD." 4 "Your medications should allow you to maintain your present diet while minimizing symptoms."

2 Alcohol increases the amount of stomach acid produced so it should be avoided. Milk may exacerbate PUD, so two to three glasses would not be recommended. There is no reason to puree or mince food, and a current diet is likely to be altered to minimize symptoms. Text Reference - p. 948

The nurse reviews a patient's medication profile and identifies that which medication has a direct, irritating effect on the gastrointestinal (GI) tract? 1 Sucralfate 2 Alendronate 3 Bethanechol 4 Metaclopromide

2 Alendronate has a direct, irritating effect that damages the mucosal barrier of the gastrointestinal (GI) tract. Sucralfate provides cryoprotection for the GI tract by forming a protective layer that serves as a barrier against acids, bile salts, and enzymes in the stomach. Bethanechol increases gastric emptying. Metaclopromide increases gastric motility and emptying. Text Reference - p. 941

After administering a dose of promethazine to a patient with nausea and vomiting, the nurse explains that what may be experienced as a common temporary adverse effect of the medication? 1 Tinnitus 2 Drowsiness 3 Reduced hearing 4 Sensation of falling

2 Although being given to this patient as an antiemetic, promethazine also has sedative and amnesic properties. For this reason, the patient is likely to experience drowsiness as an adverse effect of the medication. Tinnitus, reduced hearing, and loss of balance are not side effects of promethazine. Text Reference - p. 926

The nurse suspects that a patient is experiencing upper gastrointestinal bleeding after undergoing endoscopic therapy. Which diagnostic test is best to confirm the nurse's suspicion? 1 Biopsy 2 Angiography 3 Laparoscopy 4 Endoscopic ultrasound

2 An angiography is used to diagnose upper gastrointestinal bleeding when the bleeding persists after endoscopy. During this procedure, a catheter is placed in the left gastric or superior mesenteric artery and is advanced until the source of the bleeding is discovered. A tissue biopsy and subsequent histologic examination are important to help diagnose stomach cancer. A laparoscopy is used to determine the peritoneal spread. An endoscopic ultrasound is used to stage the disease. Text Reference - p. 954

The nurse is providing instructions to a patient who has been treated for gastrointestinal bleeding and is planned for discharge. Which statement made by the patient indicates effective learning? 1 "I should take low doses of aspirin." 2 "I should take aspirin with omeprazole." 3 "I should read the labels of over-the-counter drugs." 4 "I should take the lansoprazole one hour after meals."

2 Aspirin is a nonsteroidal antiinflammatory drug (NSAID) that reduces the prostaglandin lining of the stomach leading to inflammation of gastric mucosa and bleeding. Therefore, aspirin, taken along with proton pump inhibitors such as omeprazole and H2-receptor blockers, can reduce the bleeding risk. Aspirin, even in the lowest dose, can cause gastrointestinal bleeding. Over-the-counter drugs contain aspirin and should not be taken, because they increase the risk of bleeding. Lansoprazole is a proton pump inhibitor, which is taken before the meals to reduce the acid secretion in the stomach. Text Reference - p. 956

A postmenopausal patient complains of back pain and burning pain in midepigastric region three hours after a meal. The medical history of the patient reveals hyperparathyroidism. Which condition does the nurse suspect in the patient? 1 Gastric ulcer 2 Duodenal ulcer 3 Zollinger-Ellison syndrome 4 Postprandial hypoglycemia

2 Back pain and burning pain in the midepigastric region two to five hours after eating indicate a duodenal ulcer. These ulcers are more common in men but increasingly occur in postmenopausal women. They are associated with disease conditions such as renal failure, pancreatitis, hyperparathyroidism, and chronic obstructive pulmonary disease. Gastric ulcers are characterized by burning pain in the left epigastrium and upper abdomen. Zollinger-Ellison syndrome is a condition that involves severe peptic ulceration and hydrochloric acid secretion. Postprandial hypoglycemia is a postoperative complication of gastrectomy that occurs two hours after eating. Symptoms include sweating, weakness, confusion, palpitations, anxiety, and tachycardia. Text Reference - p. 944

Which medication does the nurse expect to be beneficial to the patient who reports difficulty swallowing and is diagnosed with achalasia? 1 Cisplatin 2 Nifedipine 3 Prednisone 4 Porfimer sodium

2 Difficulty in swallowing is known as dysphagia. It is the most common symptom of achalasia. Calcium channel blockers such as nifedipine, when administered sublingually 30 to 45 minutes before meals, may help reduce dysphagia in patients with achalasia. Cisplatin is an antineoplastic agent used in the treatment of esophageal cancer. Prednisone is used in the treatment of eosinophilic esophagitis. Porfimer sodium is a photosensitizer that is injected intravenously during photodynamic therapy of esophageal cancer. Text Reference - p. 940

Which intervention should the nurse perform when finding that a postgastrectomy patient has greenish yellow discharge eight hours after insertion of a nasogastric tube? 1 Remove the nasogastric tube. 2 Document it as a normal finding. 3 Notify the primary health care provider. 4 Place the patient in a semi-Fowler's position.

2 Discharge of bloody, greenish to yellow drainage from the nasogastric tube for 8 to 12 hours after insertion is a common observation. Therefore, the nurse should document it as a normal finding. The nurse should not remove the nasogastric tube without consulting the primary health care provider. Notifying the primary health care provider is not necessary, because discharge of bloody drainage is a normal finding. Placing the patient in a semi-Fowler's position will help prevent the risk of aspiration; however, it will not reduce the drainage from the nasogastric tube. Text Reference - p. 939

The nurse is caring for a patient treated with intravenous (IV) fluid therapy for severe vomiting. As the patient recovers and begins to tolerate oral intake, the nurse understands that which food choice would be most appropriate? 1 Iced tea 2 Dry toast 3 Hot coffee 4 Plain hamburger

2 Dry toast or crackers may alleviate the feeling of nausea and prevent further vomiting. Water is the initial fluid of choice. Extremely hot (hot coffee) or cold liquids (iced tea) and fatty foods (hamburgers) generally are not well tolerated. Text Reference - p. 928

Why does a primary health care provider place stents endoscopically while preparing a patient with esophageal cancer for surgery? 1 To prevent regurgitation 2 To improve nutrition status 3 To improve gastric emptying 4 To prevent respiratory complications

2 Endoscopic stents are placed to improve nutrition and hydration. Placing the patient in an upright position will help prevent regurgitation and improve gastric emptying. Using an incentive spirometer will help prevent respiratory complications. Text Reference - p. 938

After reviewing the medical reports of a patient with portal hypertension, the nurse concludes that the patient has esophageal varices. Which other finding supports the nurse's conclusion? 1 Kyphosis 2 Liver cirrhosis 3 Throat lacerations 4 Squamous cell cancer

2 Esophageal varices are the dilated, tortuous veins that occur in the lower portion of the esophagus. They occur as a result of portal hypertension. Esophageal varices are common complications of liver cirrhosis. Therefore, liver cirrhosis is a finding that supports the nurse's conclusion. Kyphosis is associated with a hiatal hernia. Throat lacerations may cause esophageal strictures, and squamous cell carcinoma is associated with achalasia. Text Reference - p. 940

Which instruction should the nurse provide to the patient with inflammation of the parotid glands? 1 "You should eat fibrous food." 2 "You should try chewing gum." 3 "You should avoid warm compresses." 4 "You should avoid sucking on hard candy."

2 Inflammation of parotid glands is known as parotitis. Parotitis is characterized by pain in the ear, exudate from the glands, erythema, and absence of saliva. Chewing gum helps stimulate salivary glands, resulting in the secretion of saliva. Therefore, the patient should chew gum. Intake of fibrous foods is beneficial for patients with gingivitis. Warm compresses are encouraged in patients with parotitis. The patient should use warm compresses and suck on hard candy. Text Reference - p. 928

A patient presents to the outpatient clinic with concerns over persistent signs and symptoms of heartburn (pyrosis). What is the most appropriate response for the nurse? 1 "Give the medication time to take effect. You will be feeling better really soon." 2 "I know it is uncomfortable for you. Have you been taking your medication as prescribed and making the necessary dietary adjustments?" 3 "Are you waking up during the night coughing and choking?" 4 "Is your throat sore?"

2 It is important to ascertain the patient's adherence to prescribed medication and dietary parameters first. Instructing the patient to allow time for the medication to take affect is not addressing the dietary aspect of care. Although a sore throat and nocturnal episodes of coughing and choking are symptoms associated with pyrosis, they are not the initial questions the nurse should ask about. Text Reference - p. 945

A patient is experiencing persistent vomiting with emesis consisting of contents from the small intestine. Which alteration in blood gas will the nurse expect to occur? 1 Metabolic alkalosis 2 Metabolic acidosis 3 Respiratory alkalosis 4 Respiratory acidosis

2 Metabolic acidosis occurs when contents of the small intestine are vomited because of the loss of sodium bicarbonate. Metabolic alkalosis occurs when there is a loss of gastric hydrochloric (HCL) acid. Sodium bicarbonate is secreted by the pancreas into the small intestines. The respiratory alkalosis or respiratory acidosis that would occur in the presence of the metabolic states would be a compensatory response. Text Reference - p. 925

The patient who is admitted with a diagnosis of diverticulitis and a history of irritable bowel disease and gastroesophageal reflux disease (GERD) has received a dose of Mylanta 30 mL orally. The nurse should evaluate its effectiveness by questioning the patient as to whether which symptom has resolved? 1 Diarrhea 2 Heartburn 3 Constipation 4 Lower abdominal pain

2 Mylanta is an antacid that contains both aluminum and magnesium. It is indicated for the relief of gastrointestinal discomfort, such as with heartburn associated with GERD. Mylanta can cause both diarrhea and constipation as a side effect. Mylanta does not affect lower abdominal pain. Text Reference - p. 934

The postoperative patient states that he or she has never taken pantoprozole in the past. The patient asks why he or she is getting this medication if the patient has never had heartburn. What is the best response by the nurse? 1 "The stress of surgery is likely to cause stomach bleeding if you do not receive it." 2 "This will reduce the amount of acid in your stomach until you can eat a regular diet again." 3 "This will prevent the heartburn that occurs as a side effect of your diabetes." 4 "This will prevent gas pains from the excess air in your small intestine."

2 Pantoprazole is a proton-pump inhibitor which decreases acid production in the stomach. It minimizes damage to the gastric mucosa while the patient is on bed rest and hospitalized after surgery. Pantoprazole will not prevent gas pains and will not prevent stomach bleeding from surgery. Heartburn is not a side effect of diabetes. Text Reference - p. 934

The nurse is caring for a patient with atrophic gastritis who has a cyanocobalamin deficiency. The nurse suspects what diagnosis? 1 Gastric ulcer 2 Stomach cancer 3 Nonsteroidal-induced gastritis 4 Upper gastrointestinal bleeding

2 Patients with atrophic gastritis will lose parietal cells because of atrophy, which causes pernicious anemia and stomach cancer. Gastric ulcers are caused by risk factors such as bile reflux, smoking, and alcohol abuse. Patients with a history of ulcer disease who use ulcerogenic drugs and anticoagulants tend to develop nonsteroidal-induced gastritis. Upper gastrointestinal bleeding is caused by drug-induced bleeding, esophageal bleeding, and systemic diseases. Text Reference - p. 941

A patient has what is suspected to be a gastric ulcer perforation. Which symptom does the nurse expect will be present? 1 Pyrosis 2 Rigid abdomen 3 Bright-red emesis 4 Clay-colored stools

2 Perforation results in spillage of gastric or duodenal contents into the upper peritoneal cavity. The patient experiences sudden upper abdominal pain because the spillage causes irritation of pain receptors in the visceral and parietal layers of the peritoneum. The body then attempts to protect the area by contracting the abdominal muscles, resulting in a rigid, boardlike abdomen. Pyrosis, a painful and burning sensation in the esophagus, just below the breastbone, is usually associated with regurgitation of gastric acid, also known as heartburn. Bright-red emesis may be present as a result of hemorrhage from the gastric ulcer but is not as common as the rigid, boardlike abdomen. Clay-colored stools are associated with hepatic disease, such as hepatitis, not gastric ulcer perforation. Text Reference - p. 945

The nurse finds that a patient taking a prescribed medication for gastroesophageal reflux disease (GERD) had a miscarriage. Which medication does the nurse expect to be responsible for the miscarriage? 1 Cholinergics 2 Prostaglandins 3 Antiulcer agents 4 Histamine (H2)-receptor blockers

2 Prostaglandins are a class of medications that cause uterine rupture in pregnancy. Uterine rupture in pregnancy leads to miscarriage. Therefore, the nurse suspects prostaglandins to be the cause of miscarriage. Cholinergics cause syncope, diarrhea, stomach cramps, and dizziness. Antiulcer agents such as sucralfate cause constipation. Headache, abdominal pain, constipation, and diarrhea are the side effects of histamine (H2)-receptor blockers. Text Reference - p. 934

A patient with gastroesophageal reflux disease reports to the nurse, "I feel like there is a hot, bitter liquid in my mouth." What does the nurse document in the patient's medical record? 1 Dysphagia 2 Regurgitation 3 Hypersalivation 4 Hyperchlorhydria

2 Regurgitation is described as a hot, bitter, or sour liquid coming into the throat or mouth. Dysphagia is difficulty swallowing food. Hypersalivation or water brash is the regurgitation of an excessive accumulation of saliva from the lower part of the esophagus. Hyperchlorhydria is the excessive secretion of acid in the stomach. Text Reference - p. 932

The nurse identifies that a patient is at risk for upper gastrointestinal bleeding based on the patient's history of what? 1 Hiatal hernia 2 Renal failure 3 Pernicious anemia 4 Pancreatic disease

2 Renal failure causes disturbances in the serum gastrin levels, resulting in abnormal mucosal barriers in the gastrointestinal tract and gastric acid secretion. These symptoms cause upper gastrointestinal bleeding. Hiatal hernia causes gastritis. Pernicious anemia causes stomach cancer. Pancreatic disease is the predisposing factor of duodenal ulcers. Text Reference - p. 954

The nurse is conducting patient teaching regarding aspirin. Which medication should the nurse teach to avoid while taking aspirin? 1 Acetaminophen 2 Celecoxib 3 Diltiazem 4 Digoxin

2 The patient should not take celecoxib while taking aspirin because the combination could increase the risk of gastrointestinal bleeding. Aspirin can be administered concurrently with acetaminophen, diltiazem, or digoxin. Text Reference - p. 954

Which is a common antireflux surgery for repair of a hiatal hernia? 1 Cryotherapy 2 Toupet fundoplication 3 Photodynamic therapy 4 Endoscopic mucosal resection

2 Toupet fundoplication is a common antireflux surgery for repair of a hiatal hernia. Cryotherapy, photodynamic therapy, and endoscopic mucosal resection are endoscopic therapies, not surgeries. Text Reference - p. 935

A patient requires a dose of promethazine for nausea. The prescription states to give 25 mg intravenous (IV) push q8h as needed. On hand is a vial labeled "100 mg/mL." How many milliliters should the nurse administer? 1 0.15 mL 2 0.25 mL 3 0.5 mL 4 1 mL

2 Using ratio and proportion, multiply 100 by x and multiply 25 × 1 to yield 100x = 25. Divide 25 by 100 to yield 0.25 mL. Text Reference - p. 926

The patient undergoing chemotherapy is nauseated and can have promethazine 12.5-25 mg intravenous (IV) push q4-6h as needed (PRN) for nausea. On hand is a vial labeled "25 mg/mL." How many milliliters should the nurse administer to give a 12.5-mg dose? 1 0.25 mL 2 0.5 mL 3 1 mL 4 5 mL

2 Using ratio and proportion, multiply 25 by x and multiply 12.5 × 1 to yield 25x = 25. Divide 25 by 12.5 to yield 0.5 mL. Text Reference - p. 926

What is the mechanism of action of vasopressin, a peptide hormone formed in the hypothalamus? 1 Tissue edema 2 Vasoconstriction. 3 Decreased hydrochloric acid secretion 4 Decreased esophageal and gastric mucosal irritation

2 Vasopressin causes vasoconstriction, which decreases pressure in the portal circulation and stops bleeding. Epinephrine causes tissue edema and pressure on the source of bleeding. Octreotide decreases hydrochloric acid by decreasing the release of gastrin. Histamine blockers decrease esophageal and gastric mucosal irritation. Text Reference - p. 955

A patient with stomach cancer presents with weight loss, dumping syndrome, and impaired wound healing. The nurse suspects that which surgical treatment may have caused the patient's symptoms? 1 Pyloroplasty 2 Total gastrectomy 3 Partial gastrectomy 4 Esophagojejunostomy

2 Weight loss, dumping syndrome, and impaired wound healing are all consequences of a total gastrectomy. Total gastrectomy involves the resection of the lower esophagus, removal of the entire stomach, and anastomosis of the esophagus to the jejunum. Pyloroplasty consists of surgical enlargement of the pyloric sphincter to facilitate the easy passage of contents from the stomach. Partial gastrectomy involves the removal of two-thirds of the stomach and anastomosis of the gastric stump to the duodenum or jejunum. Esophagojejunostomy is the anastomosis of the esophagus with the jejunum. Text Reference - p. 955

While caring for a postoperative patient with a nasogastric tube, the nurse suspects that the tube is repositioned. What is the priority nursing intervention to prevent complications in the patient? 1 Inserting a new nasogastric tube 2 Notifying the primary health care provider immediately 3 Irrigating the nasogastric tube with normal saline solution 4 Monitoring for the symptoms of edema and inflammation

2 When the nurse suspects repositioning of the nasogastric tube, the primary health care provider should be called as soon as possible, because there is a danger of either perforation of the gastric mucosa or a disruption of the suture line. The nurse should not insert a new tube without the order of the primary health care provider. Irrigating the tube with normal saline solution is helpful in preventing tube clogging. Monitoring for the symptoms of edema and inflammation will put the safety of the patient at risk. Text Reference - p. 950

The nurse is giving care guidelines to the caregiver of a patient with candidiasis. During the follow-up visit, the nurse finds that there the patient's condition has improved. Which actions of the caregiver does the nurse anticipate are responsible for improving the patient's condition? Select all that apply. 1 The caregiver gives the patient spicy food. 2 The caregiver applies warm compresses. 3 The caregiver gives the patient a soft and nutritious diet. 4 The caregiver avoids giving the patient lemons, tomatoes, and oranges. 5 The caregiver encourages the patient to use an antiseptic mouthwash frequently.

3 4 5 Candidiasis is a fungal infection of the mouth. The tongue, cheeks, and mouth are covered by yeast, and the mouth is sore. A patient with candidiasis has difficulty swallowing. Therefore, providing soft and nutritious food is beneficial. Lemons, oranges, and tomatoes contain citric acid, which aggravate the soreness. Therefore, avoiding citrus fruits is helpful. Using antiseptic mouthwash kills the viable candida, and is therefore beneficial to the patient. Spicy food should be avoided in patients with candidiasis, because spices irritate the ulcers and may worsen the patient's condition. Warm compresses are given if the patient has inflammation. Hence, this intervention is not beneficial to the patient. Text Reference - p. 928

A patient with malnutrition is diagnosed with stomach cancer. The nurse should question which items that are listed as treatment options for the patient? Select all that apply. 1 Billroth I 2 Billroth II 3 Chemotherapy 4 Radiation therapy 5 Total gastrectomy with esophagojejunostomy

3 4 A malnourished patient may be unresponsive to chemotherapy and radiation therapy. Malnutrition reduces the effectiveness of chemotherapy and radiation therapy, lowers overall quality of life, and lowers chances of the patient's survival. Billroth I involves pylorus removal and anastomosis of the gastric stump to the duodenum; the effectiveness of a Billroth I procedure is not affected by malnourishment. Billroth II involves connecting the greater curvature of the stomach to the jejunum; the effectiveness of a Billroth II procedure is not affected by malnourishment. A total gastrectomy with esophagojejunostomy is used to diagnose lesions located in the fundus; the effectiveness of a total gastrectomy with esophagojejunostomy is not affected by malnourishment. Text Reference - p. 952

Which diagnostic or surgical procedure can best determine the spread of stomach cancer to the peritoneum? 1 Biopsy 2 Ultrasound 3 Laparoscopy 4 Computed tomography (CT)

3 A laparoscopy can determine the spread of cancer to the peritoneum. A biopsy of the tissue and subsequent histologic examination are important for the diagnosis of stomach cancer. Ultrasound and CT studies are used to stage the disease. Text Reference - p. 952

A patient reports abdominal pain, weight loss, indigestion, and a feeling of fullness in stomach. Diagnostic test results indicate atrophic gastritis. Based on the assessment findings, the nurse suspects what diagnosis? 1 Achalasia 2 Gastric ulcer 3 Stomach cancer 4 Upper gastrointestinal bleeding

3 A patient with atrophic gastritis has an increased risk of developing stomach cancer, which is characterized by abdominal pain, weight loss, indigestion, and feeling of fullness in the stomach. Achalasia is characterized by substernal chest pain, dysphasia, and nocturnal regurgitation. A gastric ulcer has symptoms of burning or gaseous pain in the epigastric region one to two hours after eating a meal. Clinical manifestations of upper gastrointestinal bleeding include hematemesis, melena, and occult bleeding. Text Reference - p. 941

The primary health care provider is severing the vagus nerve in a patient with a peptic ulcer that is unresponsive to medical management. Which surgical procedure should the nurse expect for this patient? 1 Billroth I 2 Billroth II 3 Vagotomy 4 Pyloroplasty

3 A vagotomy is severing of the vagus nerve either totally (truncal) or selectively (highly selective vagotomy). These procedures are done to decrease gastric acid secretion. Billroth I involves a partial gastrectomy with the removal of two-thirds of the stomach and the anastomosis of the gastric stump to the duodenum. Billroth II is partial gastrectomy with the removal of two-thirds of the stomach and the anastomosis of the gastric stump to the jejunum. Pyloroplasty consists of surgical enlargement of the pyloric sphincter to facilitate easy passage of contents from the stomach. Text Reference - p. 949

The nurse is caring for a patient who presents with abdominal pain, rigid board-like abdomen and shallow grunting respirations after undergoing a surgery. Which surgery does the nurse expect to be beneficial for the patient? 1 Vagotomy 2 Endoscopy 3 Laparoscopy 4 Pyloroplasty

3 Abdominal pain, rigid board-like abdomen, and shallow grunt respirations are the clinical manifestations of perforation. Laparoscopic suture closure is performed, either with or without an omental patch, to cover the repaired ulcer and decrease the probability of leakage. A vagotomy is the severing of the vagus nerve, either totally or selectively, in order to decrease gastric acid secretion. An endoscopy is performed for gastric outlet obstruction to determine active ulcers. Pyloroplasty consists of the surgical enlargement of the pyloric sphincter to facilitate the easy passage of contents from the stomach. Text Reference - p. 948

The patient is having a gastroduodenostomy (Billroth I operation) for stomach cancer. What long-term complication is occurring when the patient reports generalized weakness, sweating, palpitations, and dizziness 15 to 30 minutes after eating? 1 Malnutrition 2 Bile reflux gastritis 3 Dumping syndrome 4 Postprandial hypoglycemia

3 After a Billroth I operation, dumping syndrome may occur 15 to 30 minutes after eating because of the hypertonic fluid going to the intestine and additional fluid being drawn into the bowel. Malnutrition may occur, but does not cause these symptoms. Bile reflux gastritis cannot happen when the stomach has been removed. Postprandial hypoglycemia occurs with similar symptoms, but two hours after eating. Text Reference - p. 953

Regurgitation is defined as: 1 A forceful expulsion of stomach contents without nausea 2 A feeling of discomfort in the epigastrium with a conscious desire to vomit 3 An effortless process in which partially digested food slowly comes up from the stomach 4 Involuntary wavelike movements occurring within the alimentary canal that force contents onward

3 An effortless process in which partially digested food slowly comes up from the stomach is the definition of regurgitation. A forceful expulsion of stomach contents without nausea is the definition of projectile vomiting. The feeling of discomfort in the epigastrium with a conscious desire to vomit is the definition of nausea. Involuntary wavelike movements occurring within the alimentary canal that force contents onward is defined as peristalsis. Text Reference - p. 925

While reviewing the laboratory reports of a patient believed to have stomach cancer, the nurse finds that the patient has vitamin B12 deficiency. Which does the nurse suspect in the patient? 1 Hiatal hernia 2 Bile reflux gastritis 3 Autoimmune gastritis 4 Gastroesophageal reflux disease (GERD)

3 Autoimmune gastritis is an inherited disorder which occurs due to loss of parietal cells, resulting in cobalamin (vitamin B12) deficiency. Hiatal hernia is herniation of a portion of the stomach into the esophagus. It is caused by a weakening of muscles in the diaphragm. Factors that may predispose a patient to a hiatal hernia are pregnancy, ascites, tumors, and obesity. Bile reflux gastritis is a postoperative disorder of gastrectomy, which occurs due to bile reflux in to the stomach. Gastroesophageal reflux disease (GERD) occurs due to an incompetent lower esophageal sphincter (LES); it is characterized by heartburn. Text Reference - p. 941

Which medication has a side effect of milk-alkali syndrome? 1 Misoprostol 2 Metoclopramide 3 Calcium carbonate 4 Aluminum hydroxide

3 Calcium carbonate is an antacid used to treat gastroesophageal reflux disease (GERD). Ingestion of calcium carbonate may result in milk-alkali syndrome. Misoprostol causes abdominal pain, diarrhea, and gastrointestinal bleeding. Central nervous system side effects such as anxiety, hallucinations, and tremors are caused by metoclopramide. Aluminum hydroxide causes constipation. Text Reference - p. 934

Which of the conditions is not a common cause of an upper gastrointestinal (GI) bleed? 1 Esophageal varices 2 Stomach cancer 3 Cholecystitis 4 Nonsteroidal antiinflammatory drugs (NSAIDS)

3 Cholecystitis affects the gastrointestinal system but is not associated with an upper GI bleed. Esophageal varices, stomach cancer, and NSAIDS are common causes of an upper GI bleed. Text Reference - p. 956

The nurse is caring for a patient who presents with burning pain in the midepigastric region five hours after eating. The patient's medical history reveals pancreatic disease. Which condition does the nurse suspect? 1 Gastric ulcer 2 Acute gastritis 3 Duodenal ulcer 4 Stomach cancer

3 Duodenal ulcers are characterized by burning pain in the midepigastric region that occurs two to five hours after eating. Gastric ulcers are characterized by burning or gaseous pressure in the high left epigastrium and upper abdomen. Symptoms of acute gastritis include anorexia, nausea, vomiting, and feeling of fullness. Signs of stomach cancer include anemia, indigestion, weight loss, abdominal pain, and satiety. Text Reference - p. 944

Which condition does the nurse suspect in a patient who reports pain and discomfort in the upper abdomen? 1 Pyrosis 2 Halitosis 3 Dyspepsia 4 Water brash

3 Dyspepsia is discomfort or pain in the upper abdomen. Pyrosis or heartburn is a burning, tight sensation that is felt intermittently beneath the lower sternum and spreads toward the throat or jaw. Halitosis is foul-smelling breath in patient with achalasia. Water brash or hypersalivation is the regurgitation of an excessive accumulation of saliva from the lower part of the esophagus. Text Reference - p. 932

A female patient has a sliding hiatal hernia. What nursing interventions will prevent the symptoms of heartburn and dyspepsia that she is experiencing? 1 Keep the patient nothing by mouth (NPO) 2 Put the bed in the Trendelenberg position 3 Have the patient eat four to six smaller meals each day 4 Give various antacids to determine which one works for the patient

3 Eating smaller meals during the day will decrease the gastric pressure and the symptoms of hiatal hernia. Keeping the patient NPO or in a Trendelenberg position are not safe or realistic for a long period of time for any patient. Varying antacids will be administered only with the health care provider's prescription, so this is not a nursing intervention. Text Reference - p. 933

Which is the first indication of gastroesophageal reflux disease and a hiatal hernia in elderly patients? 1 Halitosis 2 Pernicious anemia 3 Esophageal bleeding 4 Epigastric tenderness

3 Esophageal bleeding is the first indication of gastroesophageal reflux disease and a hiatal hernia in elderly patients. Halitosis is an indication of achalasia. Pernicious anemia and epigastric tenderness are indications of gastritis. Text Reference - p. 937

Which condition is indicated by endoscopic findings of saclike pouches on the esophagus? 1 Esophageal varices 2 Esophageal stricture 3 Esophageal diverticula 4 Eosinophilic esophagitis

3 Esophageal diverticula are saclike outpouchings formed on one or more layers of the esophagus. Esophageal varices are dilated and tortuous veins observed in the lower portion of the esophagus due to portal hypertension. Narrowing of the esophagus is called esophageal stricture. Eosinophilic esophagitis is characterized by swelling of the esophagus. Text Reference - p. 939

The nurse determines that a patient has experienced the beneficial effects of therapy with famotidine when which symptom is relieved? 1 Nausea 2 Belching 3 Epigastric pain 4 Difficulty swallowing

3 Famotidine is an H2-receptor antagonist that inhibits parietal cell output of HCl acid and minimizes damage to gastric mucosa related to hyperacidity, thus relieving epigastric pain. Famotidine is not indicated for nausea, belching, and dysphagia. Text Reference - p. 934

Linked to stomach cancer and non-Hodgkin's lymphoma, what is a common cause of gastritis? 1 Syphilis 2 Cytomegalovirus 3 Helicobacter pylori 4 Mycobacterium species

3 H. pylori infection is highest in underdeveloped countries and in people of low socioeconomic status. Infection likely occurs during childhood with transmission from family members to the child, possibly through a fecal-oral or oral-oral route. Syphilis, cytomegalovirus, and Mycobacterium species also are causes of chronic gastritis, but are not as common as H. pylori. Text Reference - p. 941

Which complication occurs when cancer erodes through the esophagus and into the aorta? 1 Choking 2 Hoarseness 3 Hemorrhage 4 Blood-flecked regurgitation

3 Hemorrhage occurs when esophageal cancer erodes through the esophagus and into the aorta. Choking and hoarseness occur when the tumor is in the upper third of the esophagus. Blood-flecked regurgitation occurs with severe esophageal stenosis. Text Reference - p. 937

The nurse is teaching a patient preventive strategies to avoid staphylococcal poisoning. Which action indicates effective learning? 1 The patient avoids eating food immediately after cooking it. 2 The patient boils canned food for five minutes before eating it. 3 The patient refrigerates leftover food to eat the next day. 4 The patient avoids cooking meat at a high temperature.

3 Immediate refrigeration of foods helps prevent staphylococcal poisoning. The nurse should instruct the patient to eat food immediately after cooking it, because it prevents the growth of microorganisms. Boiling canned food for 15 minutes before eating prevents botulism. Cooking meat at a high temperature prevents food poisoning. Text Reference - p. 958

The nurse is caring for a patient with upper gastrointestinal (GI) bleeding caused by acute hemostasis. Which medication is beneficial to this patient? 1 Etoposide 2 Octreotide 3 Epinephrine 4 Vasopressin

3 Injection therapy with epinephrine is used to treat gastrointestinal (GI) bleeding caused by acute hemostasis. Epinephrine causes tissue edema and pressure on the source of the bleeding. Etoposide is used in the chemotherapy of stomach cancer. Octreotide and vasopressin are used to treat upper GI bleeding caused by esophageal varices. Text Reference - p. 956

The nurse is connecting a nasogastric tube (NG) to suction for a patient who underwent abdominal surgery. Which therapy should be maintained in this condition? 1 Adjuvant therapy 2 Triple-drug therapy 3 Intravenous therapy 4 Cobalamin replacement therapy

3 Intravenous (IV) therapy includes adding potassium and vitamin supplements to the infusion until oral feedings are resumed. Adjuvant therapy includes surgical resection and chemotherapy for stomach cancer. Triple-drug therapy that includes proton pump inhibitor, amoxicillin, and clarithromycin is prescribed for Helicobacter pylori infection. Cobalamin replacement therapy is prescribed for pernicious anemia. Text Reference - p. 951

Which respiratory complication occurs due to irritation of the upper airway by gastric secretions? 1 Asthma 2 Pneumonia 3 Laryngospasm 4 Chronic bronchitis

3 Laryngospasm occurs due to irritation of the upper airway by gastric secretions. Asthma, pneumonia, and chronic bronchitis are respiratory complications that occur due to aspiration. Text Reference - p. 932

On a follow-up visit, the nurse finds that a patient prescribed prokinetic medication to treat nausea and vomiting has discontinued the medication. Which complication does the nurse expect to be present? 1 Xerostomia 2 Somnolence 3 Tardive dyskinesia 4 Elevated liver function tests

3 Metoclopramide and domperidone are the prokinetic agents used to treat vomiting and nausea. When metoclopramide is discontinued, tardive dyskinesia occurs. It is characterized by involuntary movements of the patient's tongue, lips, face, trunk, and extremities. Xerostomia and somnolence are side effects of anticholinergic medications. Serotonin (5-HT3) antagonists produce elevated liver function tests as a side effect. Text Reference - p. 925

A patient reports pain in the upper abdomen after eating. Upon reviewing the medical reports, the nurse finds that the patient underwent pylorus surgery. Which medication does the nurse expect the primary health care provider to prescribe? 1 Fluticasone 2 Misoprostol 3 Cholestyramine 4 Isosorbide dinitrate

3 Pain in the upper abdomen after eating is known as epigastric distress. It occurs due to reflux of bile into the stomach in patients who underwent gastric surgery involving the reconstruction or removal of the pylorus. Cholestyramine is a bile acid sequestrant that binds bile in the gastrointestinal tract to prevent its reabsorption. Fluticasone is a corticosteroid used to treat eosinophilic esophagitis. Misoprostol is used to treat gastric ulcers caused by NSAIDs and aspirin. Isosorbide dinitrate is used to treat dysphagia. Text Reference - p. 950

The nurse is assessing a patient who has suspected esophageal cancer. Which of these is the most common symptom of esophageal cancer? 1 Weight loss 2 Regurgitation 3 Progressive dysphagia 4 Epigastric pain during swallowing.

3 Progressive dysphagia is the most common symptom of esophageal cancer and may be described as a substernal feeling, as if food were not passing. Initially, the dysphagia occurs only with meat, then with soft foods, and eventually with liquids. Weight loss, regurgitation, and epigastric pain during swallowing are also symptoms of esophageal cancer, but occur later and are not the most common symptom. Text Reference - p. 937

Which medication increases gastric motility and gastric emptying? 1 Droperidol 2 Scopolamine 3 Domperidone 4 Dexamethasone

3 Prokinetic agents such as domperidone increase gastric motility and gastric emptying. Droperidol is a butyrophenone that blocks neurochemicals triggering nausea and vomiting. Scopolamine is an anticholinergic that blocks cholinergic pathways to the vomiting center, but does not increase gastric motility and emptying. Dexamethasone is a corticosteroid used in the management of acute and delayed chemotherapy-induced vomiting. Text Reference - p. 926

The nurse identifies that radiation therapy is used concurrently with chemotherapy for cancer treatment for what reason? 1 To stage the disease 2 To examine histology 3 To reduce tumor mass 4 To determine peritoneal spread

3 Radiation therapy is concurrently used with chemotherapy to reduce recurrence and as a palliative measure to decrease tumor mass. This combination therapy provides temporary relief of obstruction. Endoscopic ultrasounds and computed tomography are used to stage the disease. Biopsies are used to perform a histologic examination. Peritoneal spread of cancer is determined by laparoscopy. Text Reference - p. 952

On reviewing the previous medical history of a patient with a sore mouth who has been on a high-dose of antibiotics for a year, the nurse finds bluish-white "milk-curd" lesions in the mouth. Which medication will the nurse expect to be beneficial? 1 Acyclovir 2 Fluticasone 3 Miconazole 4 Tetracycline

3 Sore mouth and bluish-white "milk-curd" lesions on the mucosa of mouth indicate oral candidiasis caused by infection with yeast-like fungus. Oral candidiasis is caused by prolonged use of high-dose antibiotics. Miconazole is an antifungal medication used in the treatment of oral candidiasis. Acyclovir is used in the treatment of herpes simplex. Fluticasone and tetracycline are used in the treatment of aphthous stomatitis. Text Reference - p. 928

The registered nurse explains a dietary plan to a postoperative patient of gastric surgery who is preparing for discharge. Which statement made by the patient indicates the need for further teaching? 1 "I should reduce my meal size." 2 "I should reduce my intake of liquids with meals." 3 "I should consume more sugar with every meal." 4 "I should consume dry foods with low carbohydrate content."

3 Taking large amounts of sugars after gastric surgery will raise the blood sugar levels because sugar is readily is absorbed into the bloodstream. As a result, the body produces more insulin leading to a drop in blood sugar levels, which causes dizziness, sweating, tachycardia, and diarrhea. Therefore, the patient should avoid consuming more sugar with every meal. The meal size should be reduced because the stomach's reservoir is diminished after surgery. Taking more amounts of liquids with meals may cause nausea and vomiting in the patient. The patient is best able to tolerate dry foods with low carbohydrate content. Text Reference - p. 950

A patient with a peptic ulcer begins vomiting. The nurse would expect and be concerned with which type of vomitus? 1 Fecal 2 Bilious 3 "Coffee ground" 4 Undigested food

3 The appearance of blood exposed to hydrochloric acid and other digestive enzymes in the stomach is dark brown with a coffee-ground consistency. This should be reported by the nurse. Fecal vomitus would be experienced with a total bowel obstruction. Bilious vomitus or undigested food may be seen with various gastrointestinal disturbances, such as gallbladder disease, gastroenteritis, or gastritis. Text Reference - p. 925

The patient is having an esophagoenterostomy with anastomosis of a segment of the colon to replace the resected portion. What initial postoperative care should the nurse expect when this patient returns to the nursing unit? 1 Turn, deep breathe, cough, and use spirometer every four hours 2 Maintain an upright position for at least two hours after eating 3 Nasogastric (NG) tube will have bloody drainage and it should not be repositioned 4 Keep in a supine position to prevent movement of the anastamosis

3 The patient will have bloody drainage from the NG tube for 8 to 12 hours and it should not be repositioned or reinserted without contacting the health care provider. Turning and deep breathing will be done every two hours and the spirometer will be used more often than every four hours. Coughing would put too much pressure in the area and should not be done. Because the patient will have the NG tube, the patient will not be eating yet. The patient should be kept in a semi-Fowler's or Fowler's position, not supine position, to prevent reflux and aspiration of secretions. Text Reference - p. 938

What are the first clinical manifestations of peptic ulcer disease that occur in elderly patients? Select all that apply. 1 Indigestion 2 Abdominal pain 3 Abdominal sounds 4 Frank gastric bleeding 5 Decrease in hematocrit

4 5 Frank gastric bleeding and a decrease in hematocrit are the first clinical manifestations of peptic ulcer disease in elderly patients. Indigestion and abdominal pain are the clinical manifestations of stomach cancer. Abdominal sounds are the clinical manifestations of dumping syndrome, a complication of peptic ulcer disease surgery. Text Reference - p. 951

The nurse is evaluating a patient after teaching about management of peptic ulcer disease. Which statement by the patient indicates the need for further teaching? Select all that apply. 1 "I should report increased vomiting or epigastric pain." 2 "I should avoid smoking, because it may delay healing of the ulcer." 3 "I should avoid spicy and acidic food that may cause epigastric distress." 4 "I should take medications that include only antisecretory class of drugs." 5 "I should take over-the-counter drugs that have ingredients like aspirin."

4 5 While teaching a patient about management of peptic ulcer disease, the nurse should instruct the patient to take medications that include both antisecretory drugs and antibiotics, because of the development of antibiotic resistance organisms; this can be reduced by using antisecretory drugs concurrently with antibiotics. Over-the-counter (OTC) drugs that contain ingredients like aspirin should be avoided, because they destroy mucosal cells and increase the risk of ulcer development. The patient should report increased vomiting or epigastric pain. The patient should avoid smoking, because it may delay ulcer healing. The patient should follow dietary modifications by avoiding spicy and acidic food that may cause epigastric distress. Text Reference - p. 949

The nurse is caring for a patient with variceal bleeding. Which procedure should be used for this patient? 1 Gastrectomy 2 Billroth I and II 3 Radiation therapy 4 Balloon tamponade

4 A balloon tamponade involves the insertion of balloons into the esophagus or stomach by endoscopy to alleviate refractory bleeding; this method should be used to treat a patient with variceal bleeding. Gastrectomy is a procedure involving the removal of the stomach, which is done in patients with stomach cancer if the lesion is located in the fundus. The Billroth I and II procedure is used for patients with stomach cancer to treat lesions located in the antrum or the pyloric region. Radiation therapy is a treatment option for patient with stomach cancer. Text Reference - p. 955

A patient reports having a dry mouth and asks for something to drink. The nurse recognizes that this symptom most likely can be attributed to a common adverse effect of which medication that the patient is taking? 1 Digoxin 2 Cefotetan 3 Famotidine 4 Promethazine

4 A common adverse effect of promethazine, an antihistamine/antiemetic agent, is dry mouth; another is blurred vision. Common side effects of digoxin are yellow halos and bradycardia. Common side effects of cefotetan are nausea, vomiting, stomach pain, and diarrhea. Common side effects of famotidine are headache, abdominal pain, constipation, or diarrhea. Text Reference - p. 926

The nurse is caring for a patient who underwent a gastrectomy. Upon clinical examination, the primary health care provider suspects a long-term complication such as pernicious anemia. Which therapy is beneficial for the patient? 1 Triple-drug therapy 2 Intravenous therapy 3 Quadruple-drug therapy 4 Cobalamin replacement therapy

4 A patient who has undergone a gastrectomy may have a long-term complication of pernicious anemia due to the loss of intrinsic factor, which is produced by parietal cells. Cobalamin replacement therapy restores the intrinsic factor and halts vitamin B12 deficiency that occurs due to pernicious anemia. Triple-drug and quadruple-drug therapy are administered for patients with long-term Helicobacter pylori (H. pylori) infection. Intravenous therapy is used when a nasogastric tube is connected to the suction. Text Reference - p. 951

A 72-year-old patient was admitted with epigastric pain caused by a gastric ulcer. Which patient assessment warrants an urgent change in the nursing plan of care? 1 Chest pain relieved with eating or drinking water 2 Back pain three or four hours after eating a meal 3 Burning epigastric pain 90 minutes after breakfast 4 Rigid abdomen and vomiting following indigestion

4 A rigid abdomen with vomiting in a patient who has a gastric ulcer indicates a perforation of the ulcer, especially if the manifestations of perforation appear suddenly. Midepigastric pain is relieved by eating, drinking water, or antacids with duodenal ulcers, not gastric ulcers. Back pain three to four hours after a meal is more likely to occur with a duodenal ulcer. Burning epigastric pain one to two hours after a meal is from an expected manifestation with a gastric ulcer related to increased gastric secretions and does not cause an urgent change in the nursing plan of care. Text Reference - p. 944

During rounds, the nurse notes that a patient who had a total gastrectomy the day before has a very small amount of fluid draining from the nasogastric (NG) tube. What is the nurse's priority action? 1 Increase the power on the suction device. 2 Irrigate the NG tube with 50 mL of sterile saline. 3 Notify the primary health care provider immediately. 4 Continue to monitor the patient and NG tube drainage.

4 After total gastrectomy, the NG tube does not drain a large quantity of secretions because removal of the stomach has eliminated the reservoir capacity. The nurse will only need to continue to monitor the patient and the NG tube drainage. Increasing the level of suction places the patient at higher risk for acid-base imbalance. Irrigating the NG tube is not necessary. The health care provider does not need to be notified, because this is a normal finding. Text Reference - p. 951

In assessing the vital signs of a patient with an upper gastrointestinal (GI) bleed, it is important to determine whether the patient is in which kind of shock? 1 Neurogenic 2 Cardiogenic 3 Septic 4 Hypovolemic

4 Although fluctuations in vital signs occur in neurogenic, cardiogenic, and septic shock states, these fluctuations are not associated with blood loss. However, signs and symptoms of hypovolemic shock caused by GI blood loss, such as elevated heart rate, respiratory rate, and decreased blood pressure, would be evident. Assessment of the patient's vital signs assist the nurse in determining whether patient is in hypovolemic shock. Text Reference - p. 956

Which antacid used in the treatment of gastrointestinal reflux disease (GERD) causes phosphorus depletion with chronic use? 1 Magnesium oxide 2 Calcium carbonate 3 Sodium bicarbonate 4 Aluminum hydroxide

4 Aluminum hydroxide is an antacid used for treating GERD that causes phosphorus depletion with chronic use. Magnesium oxide causes diarrhea and hypermagnesemia. Calcium carbonate causes renal calculi, diarrhea, milk-alkali syndrome, and constipation. Sodium bicarbonate causes milk-alkali syndrome when used with large amounts of calcium. Text Reference - p. 934

Which complication does the nurse expect in a patient with gastroesophageal reflux disease (GERD) who has been taking dexlansoprazole for one year? 1 Syncope 2 Dyskinesia 3 Renal calculi 4 Bone fracture

4 Dexlansoprazole is a proton pump inhibitor (PPI). PPIs inhibit the proton pump responsible for the secretion of hydrochloric acid, which is an important mediator of calcium absorption. Long-term use of this medication will increase the risk of a bone fracture. Syncope is the temporary loss of consciousness leading to fainting. It occurs as a side effect of cholinergic drugs. Dyskinesia is a side effect of prokinetic medications. Renal calculi are a side effect of antacids. Text Reference - p. 934

A patient presents with severe abdominal burns. Diagnostic tests reveal a discrete, deep ulcer in the fundus of the stomach. The nurse expects that the patient will be diagnosed with what? 1 Gastric ulcer 2 Duodenal ulcer 3 Stomach cancer 4 Stress-related mucosal disease (SRMD)

4 Diffuse superficial mucosal injury or discrete deeper ulcers in the fundus and body portions of the stomach indicate stress-related mucosal disease (SRMD), which may occur as a complication of major trauma or burns. Gastric ulcers are characterized by burning or gaseous pain in the epigastric region. Duodenal ulcers are characterized by burning or cramplike pain in mid-epigastric region. Stomach cancer is characterized by early satiety, abdominal pain, and weight loss. Text Reference - p. 954

Where does Zenker's diverticulum occur? 1 At the wall of the esophagus 2 Near the esophageal midpoint 3 At the lower portion of the esophagus 4 Above the upper esophageal sphincter

4 Diverticula are the saclike outpouchings of one or more layers of the esophagus. Zenker's diverticulum occurs above the upper esophageal sphincter. Esophageal cancers occur at the wall of the esophagus. Traction diverticulum occurs near the esophageal midpoint. Esophageal varices occur at the lower portion of the esophagus. Text Reference - p. 939

Which diagnostic study is used to stage esophageal cancer? 1 Radionuclide tests 2 Manometric studies 3 Esophagram (barium swallow) 4 Endoscopic ultrasonography (EUS)

4 EUS is used to stage esophageal cancer. Radionuclide tests help detect reflux of gastric contents. Manometric studies help measure pressure in the esophagus, lower esophageal sphincter, and esophageal motility function. Esophagram (barium swallow) shows the narrowing of the esophagus at the tumor site. Text Reference - p. 937

Which procedure involves the disruption of the lower esophageal sphincter using balloons? 1 Heller myotomy 2 Esophagectomy 3 Esophagoenterostomy 4 Endoscopic pneumatic dilation

4 Endoscopic pneumatic dilation is the procedure in which the lower esophageal sphincter is disrupted using balloons of increasing diameter. A Heller myotomy is a surgical procedure in which the lower esophageal sphincter is disrupted laparoscopically. Esophagectomy is the removal of part or the entire esophagus. Esophagoenterostomy is the resection of a portion of the esophagus and anastomosis of a segment of the colon to the remaining portion. Text Reference - p. 940

When administering a dose of ondansetron, the nurse would teach the patient to report which common adverse effect? 1 Double vision 2 Paresthesias 3 Nausea 4 Headache

4 Headache that is severe enough to require an analgesic medication is a common adverse effect of ondansetron. The patient should be taught to report this symptom to the nurse. Double vision and paresthesias are not adverse reactions associated with ondansetron. Nausea would indicate the ondansetron was not effective. Text Reference - p. 926

The nurse is caring for a patient with chronic gastritis. Which of these symptoms is associated specifically with this condition? 1 Severe diarrhea 2 Gastric hemorrhage 3 Nausea and vomiting 4 Cobalamin deficiency

4 In chronic gastritis the manifestations are similar to those described for acute gastritis. Patients with acute gastritis and chronic gastritis may experience nausea and vomiting or hemorrhage. However, with chronic gastritis, when the parietal cells are lost as a result of atrophy, the source of intrinsic factor is also lost. The loss of intrinsic factor, a substance essential for the absorption of cobalamin in the terminal ileum, ultimately results in cobalamin deficiency. With time, the body's storage of cobalamin in the liver becomes depleted, and a state of deficiency exists. Diarrhea is not associated with gastritis. Text Reference - p. 941

Which antacid does the nurse expect the primary health care provider to prescribe to a patient who reports mild, intermittent chest pain after meals? 1 Sodium citrate 2 Calcium carbonate 3 Aluminum carbonate 4 Magnesium trisilicate

4 Magnesium trisilicate helps relieve heartburn after meals by creating a foam barrier on the stomach acid. It is prescribed for mild and intermittent chest pain after meals. Sodium citrate, calcium carbonate, and aluminum carbonate are also antacids used in the treatment of gastroesophageal reflux disease causing heartburn; however, they are not used for mild and intermittent heartburn. Text Reference - p. 934

Which medication for peptic ulcer disease causes hallucinations? 1 Nizatidine 2 Sucralfate 3 Omeprazole 4 Metoclopramide

4 Metoclopramide is a prokinetic agent that causes central nervous system side effects such as hallucinations and anxiety. Nizatidine is a histamine (H2)-receptor blocker that causes abdominal pain, headache, diarrhea, and constipation. Sucralfate is an antiulcer medication that causes constipation. Omeprazole is a proton pump inhibitor that causes nausea, abdominal pain, headache, diarrhea, and flatulence. Text Reference - p. 934

The nurse identifies that a patient is at risk for upper gastrointestinal bleeding based on the patient's history of taking medications in what drug class? 1 Antacids 2 Anticholinergics 3 Tricyclic antidepressants 4 Nonsteroidal anti-inflammatory drugs (NSAIDs)

4 Nonsteroidal antiinflammatory drugs (NSAIDs) may disrupt the production of prostaglandins, which are required for the protection of the stomach's mucosal lining, and result in upper gastrointestinal bleeding. Antacids are used as an adjunct therapy for the treatment of peptic ulcers. Anticholinergic drugs are used occasionally to treat peptic ulcer disease. Tricyclic antidepressants are used to reduce gastric acid secretions associated with peptic ulcer disease. Text Reference - p. 954

Which technique should the nurse use to effectively administer a dose of promethazine by the intramuscular (IM) route? 1 Numb the area with ice before injection. 2 Administer in the flank area to increase absorption. 3 Inject at a 45-degree angle. 4 Use the Z-track technique

4 Promethazine can be irritating to tissues; therefore, the medication should be injected into the upper outer quadrant of the buttock with the use of the Z-track technique. It is not required to numb the area before injection. This medication should not be administered subcutaneously in the flank because of irritation to tissues. Intramuscular injections always should be administered at a 90-degree angle. Text Reference - p. 925

Which medication is beneficial to a patient reporting coughing, dyspnea, and radiating pain to the back, neck, and jaw? 1 Nifedipine 2 Prednisone 3 Isosorbide dinitrate 4 Aluminum hydroxide

4 Respiratory symptoms such as coughing and dyspnea accompanied with radiating pain to the back, neck, and jaw indicate gastroesophageal reflux disease (GERD). GERD-related chest pain is similar to angina. Antacids such as aluminum hydroxide are used in the treatment of GERD-related chest pain. Nifedipine and isosorbide dinitrate are used in the treatment of achalasia. Prednisone is used in the treatment of eosinophilic esophagitis. Text Reference - p. 934

The nurse reviews a patient's medical record and notes a history of traumatic injuries as a result of a car fire that occurred one month prior to the current admission. The nurse recognizes that the patient is at risk for what condition? 1 Gastritis 2 Duodenal ulcer 3 Stomach cancer 4 Stress-related mucosal disease (SRMD)

4 Severe trauma and burns can lead to stress-related mucosal disease (SRMD). Burns cause hypersecretion of gastric acid and mucosal damage, resulting in SRMD. Shock, renal failure, and hiatal hernia are the risk factors for gastritis. Duodenal ulcers are caused by chronic obstructive pulmonary disease, pancreatic disease, and chronic renal failure. Atropic gastritis, pernicious anemia, and adenomatous polyps cause stomach cancer. Text Reference - p. 954

A patient is on nonsteroidal antiinflammatory drugs (NSAIDs). Which complication does the nurse anticipate? 1 Achalasia 2 Duodenal ulcer 3 Stomach cancer 4 Silent peptic ulcer

4 Silent peptic ulcers show no symptoms of ulcer disease until the presentation of their final, fatal illness. The ulcers occur in older patients or patients who take NSAIDs. Achalasia is a primary motility disorder characterized by the absence of peristalsis and impaired relaxation of the lower esophageal sphincter (LES) in response to swallowing. Duodenal ulcers occur due to excessive alcohol ingestion and smoking that result in increased hydrochloric acid secretion. Stomach cancer begins with a nonspecific mucosal injury as a result of infection, autoimmune-related inflammation, and tobacco use. Text Reference - p. 944

A patient underwent abdominal surgery four days ago and has sutures in the upper epigastric region. Which is the most appropriate initial nursing intervention to prevent pulmonary complications? 1 Administer steam inhalation. 2 Administer bronchodilator drugs. 3 Encourage early ambulation. 4 Instruct the patient to splint the incision site with a pillow while coughing.

4 Splinting the incision site with a pillow reduces the pain during coughing and deep breathing and should be taught first. Steam inhalation and bronchodilator drugs also prevent pulmonary complications but are more useful if the patient is not able to remove pulmonary secretions by himself. Early ambulation is also a measure to prevent pulmonary complications but is not applicable in the early phase of care. Text Reference - p. 951

A patient who underwent surgical therapy for peptic ulcer disease complains of sweating, weakness, mental confusion, and increased heart rate two hours after eating. Which complication does the nurse anticipate? 1 Achalasia 2 Bile reflux gastritis 3 Dumping syndrome 4 Postprandial hypoglycemia

4 Sweating, weakness, mental confusion, and increased heart rate are clinical manifestations of postprandial hypoglycemia that occur as a complication of peptic ulcer surgery. Achalasia is a primary motility disorder characterized by the absence of peristalsis and impaired relaxation of the lower esophageal sphincter (LES) in response to swallowing. Bile reflux gastritis is a complication of abdominal surgery of the pylorus and is characterized by epigastric distress. In dumping syndrome, the patient has sweating, palpitations, and dizziness. Text Reference - p. 950

A nurse assesses a patient with suspected peptic ulcer disease. Which symptom will the patient most likely report? 1 Vomiting after meals 2 Abdominal distention after eating 3 Intolerance of fatty and spicy foods 4 Epigastric discomfort relieved by eating

4 Symptoms of peptic ulcer disease (PUD) are variable and often absent. However, discomfort, if present, may occur before meals or two to three hours after meals and at bedtime. The discomfort may be relieved by eating because the food will dilute and buffer gastric acid. Although vomiting or abdominal distention after meals may occur, they are less likely to be associated with PUD than is the relief caused by eating. Text Reference - p. 942

A patient is admitted to the hospital with a severe duodenal ulcer. The patient suddenly complains of severe pain spreading over the entire abdomen, likely due to a perforation. What should be the most immediate intervention by the nurse, if prescribed? 1 Administer nitrates. 2 Administer pain medication. 3 Prepare for laparoscopic surgery. 4 Insert a nasogastric (NG) tube into the stomach.

4 The immediate focus of management for a patient with a perforation is to stop the spillage of gastric or duodenal contents into the peritoneal cavity and restore blood volume. An NG tube is inserted into the stomach to provide continuous aspiration and gastric decompression to stop spillage through the perforation and thereby prevent peritonitis. Administering nitrates to such a patient will not be helpful in relieving the condition. Administration of pain medications and preparations for laparoscopic surgery are done later. Text Reference - p. 947

Which action of the nurse is appropriate if, while administering fluids through a jejunostomy feeding tube to a patient who underwent an esophageal surgery, the nurse suspects that the tube is misplaced? 1 Reinsert the tube. 2 Reposition the tube. 3 Document the findings and continue to administer. 4 Notify the primary health care provider immediately.

4 The nurse should notify the primary health care provider immediately if the tube is misplaced. The nurse should not reinsert or reposition the tube without an order from the primary health care provider. Documenting the findings and continuing to administer will put the patient's safety at risk. Text Reference - p. 939

The nurse is teaching a group of high school students about the prevention of food poisoning. Which comment by the student shows understanding of foodborne illness protection? 1 "We like to mix up the ingredients so the flavors will melt before we cook our beef stew." 2 "For a snack, I like to eat raw cookie dough from the package instead of baking the cookies." 3 "We only have one cutting board, so we cut up our chicken and salad vegetables at the same time." 4 "When they gave me a pink hamburger I sent it back and asked for a new bun and clean plate."

4 The student who did not accept the pink hamburger and asked for a new bun and clean plate understood that the pink meat may not have reached 160o F and could be contaminated with bacteria. Mixing ingredients and leaving them long enough for the flavors to melt, eating raw cookie dough from a refrigerated package, and only using one cutting board without washing it with hot soapy water between the chicken and salad vegetables could all lead to food poisoning from contamination. Text Reference - p. 957

The nurse provides education to a group of student nurses about preoperative preparations for bowel surgery. Which statement made by a student nurse indicates the need for further teaching? 1 "Administer enemas to cleanse the bowel." 2 "Correct malnourishment in the patient prior to surgery." 3 "Administer antibiotics to reduce the intestinal bacteria." 4 "Transfuse packed red blood cells (RBC) to correct anemia."

4 Transfusion of packed red blood cells (RBCs) to correct anemia is the preoperative preparation for stomach cancer, not bowel surgery. Administration of enemas for cleansing the bowel, correcting the malnourishment in the patient prior surgery, and administering antibiotics to reduce the intestinal bacteria are preoperative preparations for bowel surgery. Text Reference - p. 956

Deep muscle injection is the preferred route of administration of this drug used to treat nausea and vomiting because of its necrotizing effect on tissue when infiltration occurs. 1 Methergine 2 Prochlorperazine 3 Metoclopramide 4 Promethazine

4 When promethazine is administered intravenously (IV), it can leak out from the vein and cause serious damage to surrounding tissue. Deep muscle injection is the preferred route of injection administration. Methergine is used to control postpartum hemorrhage. Prochlorperazine and metoclopramide are not reported to have as damaging an effect on tissue when administered IV. Text Reference - p. 925

Which condition can be diagnosed by measuring fasting serum gastrin levels? 1 Acute gastritis 2 Pyloric obstruction 3 Gastric outlet obstruction 4 Zollinger-Ellison syndrome

4 Zollinger-Ellison syndrome can be diagnosed by measuring fasting serum gastrin levels, which will determine the amount of gastrin secreted by G cells of the stomach. Acute gastritis is diagnosed by endoscopic examination with biopsy. Pyloric obstruction is diagnosed by performing an endoscopy with dilated balloons. Gastric outlet obstruction is diagnosed by performing a barium contrast study. Text Reference - p. 945

Which factors increase intraabdominal pressure and lead to the development of a hiatal hernia? Select all that apply. 1 Ascites 2 Obesity 3 Pregnancy 4 Fatty foods 5 Peppermint

Factors increasing intraabdominal pressure include ascites, obesity, and pregnancy. They may lead to the development of a hiatal hernia. Fatty foods and peppermint are factors that decrease lower esophageal sphincter pressure. Text Reference - p. 936

Which treatment strategy does the nurse expect to be beneficial for an elderly patient with a hiatal hernia who has a cardiovascular complication? 1 Antireflux surgery 2 Laparoscopic surgery 3 Calcium channel blocker therapy 4 Nonsteroidal antiinflammatory drugs (NSAIDs)

AQ says# 2 but seems like should be #3. Laparoscopic surgery is minimally invasive and when performed in elderly patients reduces the risk of complications and time for recovery and is beneficial. Antireflux surgery is a surgical intervention and is not recommended in elderly patients with cardiovascular and pulmonary complications. Calcium channel blocker therapy decreases lower esophageal pressure in elderly patients. Nonsteroidal antiinflammatory drugs (NSAIDs) may irritate the esophageal mucosa, which may aggravate the symptoms more. Text Reference - p. 937

A registered nurse is evaluating the medication charts of four patients that have been prepared by a student nurse. Which patient's medication chart needs correction?

Gastritis is an inflammation of the gastric mucosa due to breakdown of the gastric mucosal barrier. It may be caused by Helicobacter pylori infection. The drug therapy for this includes H2-receptor blockers such as omeprazole, lansoprazole, and esomaprazole and proton pump inhibitors (PPIs) such as cimetidine, famotidine, and ranitidine. Therefore, the medication chart for patient 1 needs correction. Isosorbide dinitrate is a nitrate derivative used in the treatment of achalasia, a condition seen in patient 2. Achalasia is the loss of peristalsis in the lower esophagus. Calcium channel blockers such as nifedipine are also used to treat achalasia. A corticosteroid, such as prednisone, is effectively used in the treatment of eosinophilic esophagitis, which is used to treat patient 3. Omeprazole is used in the treatment of gastric esophageal reflux disease; this is seen in patient 4. Text Reference - p. 937


Set pelajaran terkait

Market failures: public goods and externalities

View Set

Chapter 13 Fill in the Blank Quiz

View Set

Intermediate Accounting Ch.3 part A

View Set

Network Administration Practice Questions 4.1.10

View Set